Kausalität und Quantenfeldtheorie [Duplikat]

Ich habe ein Problem mit dem Beweis der Kausalität in Peskin & Schroeder, An Introduction to QFT, Seite 28. Um Verwirrung zu vermeiden, verwende ich die Drei-Vektor-Notation und schreibe die Gl. (2.53) für j = 0 folgendermaßen:

[ ϕ ( X , T ) , ϕ ( 0 , 0 ) ] = D 3 P ( 2 π ) 3 1 2 P 2 + M 2 ( e ich P . X ich T P 2 + M 2 e ich P . X + ich T P 2 + M 2 )

Das Buch geht weiter darüber, wie der Integrand, der Lorentz-invariant ist, dieses Integral zu Null für das x aus dem Lichtkegel macht. Aber ich (ich bin kein Experte für spezielle Relativitätstheorie) möchte es strenger sehen:

nach dem Ändern von Variablen P P im ersten Term vereinfacht sich die Gleichung zu:

[ ϕ ( X , T ) , ϕ ( 0 , 0 ) ] = D 3 P ( 2 π ) 3 2 ich 2 P 2 + M 2 e ich P . X Sünde ( T P 2 + M 2 )

mit sphärischen Koordinaten:

[ ϕ ( X , T ) , ϕ ( 0 , 0 ) ] = D P D ϕ D θ P 2 Sünde θ ( 2 π ) 3 ich P 2 + M 2 e ich P X cos θ Sünde ( T P 2 + M 2 ) [ ϕ ( X , T ) , ϕ ( 0 , 0 ) ] = 0 D P P ( 2 π ) 2 2 ich X P 2 + M 2 Sünde ( P X ) Sünde ( T P 2 + M 2 )

erneut nach einer erneuten Variablenänderung u = P 2 + M 2 ,

[ ϕ ( X , T ) , ϕ ( 0 , 0 ) ] = 2 ich X M D u ( 2 π ) 2 Sünde ( X u 2 M 2 ) Sünde ( T u )

Ich kann nicht sehen, wie dieses Integral Null sein sollte X > T !!! Kann mir das bitte jemand erklären?

Antworten (2)

Ich werde auf Ihren Punkt eingehen, warum das Integral Lorentz-invariant ist, da ich aus den Kommentaren zu cdustons Antwort denke, dass dies Ihr Knackpunkt ist:

Sie können die Beziehung zwischen einer offensichtlich Lorentz-invarianten Form wie dieser sehen

D 4 P e ich P X ( P 2 M 2 )       ( 1 )
und die nicht so offensichtlich Lorentz-invariante Form
D 3 P 1 E P e ich P X       ( 2 )
unter Verwendung der Identität
1 ( P 2 M 2 ) = 1 2 E P { 1 ( E P + P 0 ) 1 ( E P P 0 ) }
Hier E P = P 2 + M 2 die On-Shell-Zeitkomponente des Impuls-Vier-Vektors ist, und P 0 ist die "generische" Zeitkomponente - nicht unbedingt auf der Shell.

Wenn Sie dies in (1) einsetzen und das tun P 0 Integral unter Verwendung der entsprechenden Kontur erhalten Sie (2).

Was eigentlich vor sich geht, wird in der Diskussion in der Nähe von Gleichung (2.40) erklärt, Sie machen ein 4-Impuls-Integral, beschränken es aber nur auf die Massenhülle mit einer Delta-Funktion. Die Beschränkung auf eine Massenschale ist eine invariante Lorentz-Operation, sodass Sie die Lorentz-Invarianz durchgehend beibehalten (obwohl es mit dem Drei-Impuls-Integral nicht so aussieht!).

Im Text sagt er, die beiden Begriffe verschwinden darunter ( X j ) ( X j ) . Mit anderen Worten, es gibt eine Lorentz-Transformation, die dauert ( X j ) ( X j ) im zweiten Term, wenn die Trennung raumartig ist ( ( X j ) 2 < 0 mit falschem Vorzeichen...). Wenn Sie das tun, verschwindet der Kommutator.

Ich stimme deinem Kommentar im Text zu, der sagt, wenn die Trennung Raum ist, kann man so eine Transformation machen und Null bekommen. Vorausgesetzt, der Term ist Lorentz-invariant. Ich kann nicht sehen, wie dieser Term Lorentz-invariant ist. Ich kann sehen, wie D 3 P F ( P ) / P 2 + M 2 ist Lorentz-invariant, aber keine Funktion wie D 3 P F ( P , X , T ) / P 2 + M 2 . Können Sie mir bitte erklären warum D 3 P e ich P . X ich T P 2 + M 2 / P 2 + M 2 ist Lorentz-invariant?
Das Ding im Exponenten ist nur das Lorentz-invariante innere Produkt zwischen dem 4-Impuls P μ = ( P , P 2 + M 2 ) und die 4-Stellung X μ = ( X , T ) (bis zu welcher Index- und Vorzeichenkonvention sie verwenden).
Das vorherige Poster ist korrekt, aber PS erwähnt dies auch ein paar Seiten früher in ihrem Text. Ich habe es gerade nicht dabei, aber schau mal.
@Blackie Eq. (2.40) auf Seite 23 zeigt, dass es Lorentz-Invariante ist.